LSAT and Law School Admissions Forum

Get expert LSAT preparation and law school admissions advice from PowerScore Test Preparation.

 Administrator
PowerScore Staff
  • PowerScore Staff
  • Posts: 8917
  • Joined: Feb 02, 2011
|
#61065
Please post your questions below!
 LSATwhizkid00
  • Posts: 3
  • Joined: Dec 28, 2018
|
#61441
I was hoping if someone here could help me clarify my reasoning for this question.

I chose D through the process of elimination. If there has been a general decline in resources devoted to workplace safety, then it can be inferred that there would be more deaths and injuries. The conclusion states that the percentage and absolute number of deaths has decreased. Since the question is asking for an AC that resolves the issue in the stimulus, D fails to do so because it doesn't explain why there have been fewer deaths.

I'm not sure if this reason alone is enough to make D the right answer.

Thanks in advance.
 Ben DiFabbio
PowerScore Staff
  • PowerScore Staff
  • Posts: 39
  • Joined: Aug 02, 2018
|
#61456
LSATwhizkid00 wrote:I was hoping if someone here could help me clarify my reasoning for this question.

I chose D through the process of elimination. If there has been a general decline in resources devoted to workplace safety, then it can be inferred that there would be more deaths and injuries. The conclusion states that the percentage and absolute number of deaths has decreased. Since the question is asking for an AC that resolves the issue in the stimulus, D fails to do so because it doesn't explain why there have been fewer deaths.

I'm not sure if this reason alone is enough to make D the right answer.

Thanks in advance.
Hi there -

Just responding to confirm that your thinking on this question is spot on. This is a resolve EXCEPT question, so any answer choice that might contradict the information we're given (i.e. deaths might increase if workplace safety resources declined) cannot resolve the issue and would therefore be the correct answer in this case. The other four answer choices offer relatively weak circumstantial explanations, but that's perfectly acceptable when we have one answer choice that would work in the opposite direction.

Keep up the good work!

- Ben

Get the most out of your LSAT Prep Plus subscription.

Analyze and track your performance with our Testing and Analytics Package.